LSAT and Law School Admissions Forum

Get expert LSAT preparation and law school admissions advice from PowerScore Test Preparation.

User avatar
 Dave Killoran
PowerScore Staff
  • PowerScore Staff
  • Posts: 5853
  • Joined: Mar 25, 2011
|
#41164
Complete Question Explanation
(The complete setup for this game can be found here: lsat/viewtopic.php?t=8477)

The correct answer choice is (E)

The question stem adds a new condition: V :longline: Z. This new rule affects the initial diagram, producing the following controlling chain:
  • pt38_o02_g1_q7.png
This condition actually simplifies the initial chain, eliminating the confusion caused by the possible movement of Z.

Answer choice (A) is incorrect because without the possibility of Z getting out of the car first, R must be the first clown to get out of the car.

Answer choice (B) is incorrect because T must exit the car no later than third.

Answer choice (C) is incorrect because the earliest Q could exit the car is fifth.

Answer choice (D) is incorrect because the latest V could get out of the car is fourth.

Answer choice (E) is the correct answer.
You do not have the required permissions to view the files attached to this post.

Get the most out of your LSAT Prep Plus subscription.

Analyze and track your performance with our Testing and Analytics Package.